LSAT and Law School Admissions Forum

Get expert LSAT preparation and law school admissions advice from PowerScore Test Preparation.

 Administrator
PowerScore Staff
  • PowerScore Staff
  • Posts: 8917
  • Joined: Feb 02, 2011
|
#80571
Complete Question Explanation

Strenghten. The correct answer choice is (A).

Answer choice (A): This is the correct answer choice.

Answer choice (B):

Answer choice (C):

Answer choice (D):

Answer choice (E):

This explanation is still in progress. Please post any questions below!
 Lauryn
  • Posts: 6
  • Joined: Jul 22, 2019
|
#80760
This question lost me. I wish I could explain my reasoning more, but I didn't like any of the answer choices. I was looking for something like "if at least some people who would never bother seeking reimbursement get the manufacturer's warranty then manufacturers will not need to reimburse many people" or "it is necessary that people who would never bother seeking reimbursement get the manufacturer's warranty so that manufacturers never need to reimburse many customers because the product did not work well or last long". Is this totally off-base? It almost seems to contradict A :hmm:
 cmorris32
  • Posts: 92
  • Joined: May 05, 2020
|
#80776
Hey Lauryn - I'm not sure if this helps but I worked on this question a lot after taking this practice test and here is how I came up with A...

The conclusion of this argument is that people should only buy a frying pan that has a manufacturer's warranty, and the author gives premises that state that the manufacturer will not offer a warranty on a product if doing so means that they will need to reimburse many customers because the product did not work well or did not last long.

My prephrase for this question was that the author was getting at the idea that the manufacturers wouldn't want to reimburse a lot of customers, so they actually will make a product work well and last long, in order to avoid reimbursing a lot of customers. This is why the author concludes that people should buy a frying pan that has a manufacturer's warranty - so I was anticipating an answer that would strengthen this idea...

A works because if most people who buy a frying pan with a warranty would seek a reimbursement if something was wrong, that strengthens the manufacturer's desire to make the plan work well and last long (again, to avoid a lot of reimbursements). That then strengthens the author's conclusion, which states that people should buy a frying pan that has a manufacturer's warranty.

Not sure if this is the right reasoning... but that is how I got to answer choice A. :-D
 Rachael Wilkenfeld
PowerScore Staff
  • PowerScore Staff
  • Posts: 1358
  • Joined: Dec 15, 2011
|
#80783
Cmorris, you are right on here---great work.

Lauryn, I'm wondering if you had the reasoning flipped around here---it sounds like you may have. Let's break it down.

Premise: Manufacturers won't offer a warranty on a product if it won't last long/work well because that would require them to pay out a lot of claims.

Conclusion: You should only buy a pan with a warranty even if you won't use it.

We need to help this argument, and although it may sound counterintuitive, we want to start by thinking through the weaknesses. The biggest thing I see here is that if no one uses the warranty, companies could offer it without requiring them to pay a lot in claims.

To strengthen the argument, we need something to say that there would at least be some people who would use the warranty.

That's what we see in answer choice (A). It goes further than the prephrase, saying that most people would use it.

Hope that helps!
User avatar
 jcg16e
  • Posts: 2
  • Joined: May 31, 2021
|
#87477
Rachael, I'm very confused about the correct answer.

Wouldn't the fact that most costumers who buy the pan with a warranty would seek reimbursement actually weaken the argument.... because if most customers who buy the pan with warranty seek reimbursement, wouldn't that mean that manufacturers will not offer a warranty because they do not want to pay reimbursements?
User avatar
 Ryan Twomey
PowerScore Staff
  • PowerScore Staff
  • Posts: 141
  • Joined: Mar 04, 2021
|
#87520
Hey jcg16e,

I'm going to post your question so I can more easily reference it:

"Wouldn't the fact that most costumers who buy the pan with a warranty would seek reimbursement actually weaken the argument.... because if most customers who buy the pan with warranty seek reimbursement, wouldn't that mean that manufacturers will not offer a warranty because they do not want to pay reimbursements?"

If most customers who buy the pan with a warranty seek reimbursements if the pan fails to work or doesn't last long, then this would discourage a manufacturer to offer a warranty.... If the pan fails to work or doesn't last long.

So this actually strengthens our argument, because it means that if manufacturers are offering warranties, that pan is more likely to be of higher quality, more likely to work well and last long, because otherwise the manufacturers would be losing money on the reimbursements.

I hope this helps explain your question and I wish you all of the luck in your studies.

Best,
Ryan
 kupwarriors9
  • Posts: 73
  • Joined: Jul 01, 2021
|
#89645
How is this a strengthen Q and not a Justify the Conclusion question
User avatar
 German.Steel
  • Posts: 55
  • Joined: Jun 12, 2021
|
#90761
I understand what makes (A) work, but (B) is also strong to me. Is "at the time of purchase" the only issue? That's not a big issue as far as I can tell, because it still strengthens, but I'm wondering if I'm missing something else. Thanks in advance
 Rachael Wilkenfeld
PowerScore Staff
  • PowerScore Staff
  • Posts: 1358
  • Joined: Dec 15, 2011
|
#90917
This is a strengthen, kupwarriors, because it's asking about what "most" strongly supports the conclusion. Remember that justify questions will not lessen the degree of justification. An answer will either justify or not. But when you see that word "most" in the question stem, you want to classify it as a strengthen question.

And German.steel, that "time of purchase" in answer choice (B) is a big big issue. The warranty isn't just to make sure the pans work well the day you buy them. The stimulus specifically says that they are also for the longevity of the pans. After all, a pan is not useful if it works great the first time you use it, but not after that.

Hope that helps!
 g_lawyered
  • Posts: 211
  • Joined: Sep 14, 2020
|
#92531
Hi P.S.,
I also thought this was a Justify Conclusion/ Sufficient Assumption. I had a different line of reasoning, I'm hoping you can advise whether I'm on the right path or not. With my reasoning, I was able to eliminate answer C-E. And was left with A & B. I eliminated B because I didn't think the "time of purchase" supported the premise because warranty means after purchasing and using for period of time. Also, the premise states that "the pan not work well or last long". By default and process of elimination, I chose A, even thought I wasn't 100% sure it was the assumption. I saw a lot of conditional reasoning (even thought I didn't see an explanation with conditional reasoning) and skipped this question during my practice test and completed it once I reviewed my results. Here's my reasoning:
Conclusion: Buy Fry Pan :arrow: Manufacture Warranty
Contrapositive: NO Manufacture Warranty :arrow: DON'T buy Fry Pan

Premise 1: Expect to give money back because doesn't work :arrow: NO Manufacture Warranty
Contrapositive: Manufacture Warranty :arrow: Give money back

I combined statements to form logical chain:
Expect to give money back because doesn't work :arrow: NO Manufacture Warranty :arrow: DON'T buy Fry Pan

Any direction would help. Thanks in advance!

Get the most out of your LSAT Prep Plus subscription.

Analyze and track your performance with our Testing and Analytics Package.